You signed in with another tab or window. Reload to refresh your session.You signed out in another tab or window. Reload to refresh your session.You switched accounts on another tab or window. Reload to refresh your session.Dismiss alert
Not absolutely sure this is a bug but can't figure out why the Holt linear trend equations produces different slope values to those given by the components() view of an ETS model. I have posted a reproducible example on Cross Validated
I am not skilled enough to look through the code, but hope that you can confirm that it should be possible to replicate the level and trend using the Holt equations.
Philip
The text was updated successfully, but these errors were encountered:
Thanks very much Rob for taking the time to respond to my question. It is a steep learning curve, and I hope you have the patience to answer one more. Having spent time rewatching and rereading chapter 8.5 and some more time reevaluating the formulas, I remain unable to calculate the first estimated slope value using the components function values and β=αβ*
Q. So if the ETS model is specified with parameters: trend(method = "A", alpha = 0.5, beta = 0.5))
is α = alpha and β* = beta ? or is β = beta ?
... to calculate the new estimated slope the equation is: bt = bt−1 + βεt - where β=αβ*
which (if β* = beta) becomes bt-1 + (0.5*0.5) * εt
If I use 0.5*0.5 I can't get the same answer as the components function. If I just use 0.5 it works. So my conclusion is β (without the *) must be equivalent to the trend parameter beta.
Not absolutely sure this is a bug but can't figure out why the Holt linear trend equations produces different slope values to those given by the components() view of an ETS model. I have posted a reproducible example on Cross Validated
https://stats.stackexchange.com/questions/630437/i-cannot-replicate-the-slope-for-an-ets-model-with-alpha-0-5-and-beta-0-5-usi
I am not skilled enough to look through the code, but hope that you can confirm that it should be possible to replicate the level and trend using the Holt equations.
Philip
The text was updated successfully, but these errors were encountered: